0 of 15 Questions completed
Questions:
You have already completed the quiz before. Hence you can not start it again.
Quiz is loading…
You must sign in or sign up to start the quiz.
You must first complete the following:
0 of 15 Questions answered correctly
Your time:
Time has elapsed
You have reached 0 of 0 point(s), (0)
Earned Point(s): 0 of 0, (0)
0 Essay(s) Pending (Possible Point(s): 0)
Average score |
|
Your score |
|
A 57-year-old male with recently diagnosed small cell lung cancer (SCLC) is presenting to the electromyography lab with a one-month history of muscle cramps and muscle pain. Percussive myotonia is appreciated on examination. Electromyography showed myokymic discharges. Autoantibodies against which of the following channels are likely present in this case?
A 3-year-old girl is admitted to the hospital with a 2-day history of constipation, lethargy, and a weak cry. Repetitive stimulation performed at 20 Hz revealed incremental response. What is the likely cause of this patient’s illness?
A 45-year-old female with a past medical history of hypertension and medication non-adherence presents to the emergency department with weakness on the left side of the body and a drooping right eye. Her blood pressure is 220/100. On physical exam, her right eye is inferiorly and laterally deviated and the pupil is non-reactive. Her strength was 3/5 on the left side. MRI brain identifies an ischemic lesion. Based on the clinical exam, which of the following arteries is most likely the cause of the patient’s symptoms?
Which of the following genetic abnormalities is most common in the pathology shown in the image below?
A 10-year-old girl is brought to the clinic with a 1-month history of progressive clumsiness. Neurological examination shows significant bilateral ataxia. Imaging was performed and showed a midline lesion in the 4th ventricle. A biopsy was performed and shown below. Based on the most likely diagnosis, what is the most appropriate treatment plan?
Which of the following is the mechanism of action of entacapone?
A 68-year-old man with left leg weakness and pain has an MRI performed to evaluate for a possible etiology. Her imaging is provided below. Based on the location of the lesion, which of the following muscles is most likely weak on examination?
A 35-year-old woman with a past medical history of ocular and oral dryness presents with a 5-month history of progressive difficulties with ambulation. On exam, there is diminished vibratory sensation along the plantar/dorsal surfaces of her feet, 1+ ankle reflexes bilaterally, and a positive Romberg sign. Strength is normal. A tissue biopsy confirms the diagnosis. Which of the following is the most likely diagnosis?
A 65-year-old male with a past medical history of diabetes, hypertension, and a 40-pack-year smoking history comes into the ED with an acute onset of ataxia, left facial numbness, right arm, and leg numbness, and dizziness. He also has intractable hiccups. A CTA angiogram shows the cause of the patient’s symptoms. Based on the clinical exam, which of the following arteries is occluded?
An autopsy was performed on a 30-week gestational age stillborn infant. An image of the brain before histopathologic analysis is shown below. Which of the following was likely also appreciated during the autopsy?
Cytotoxic edema would be best appreciated on which of the following MRI sequences?
Which of the following dermatomes are correctly matched with their associated anatomical location?
A 21-year-old male with a history of cirrhosis, dysarthria, and personality changes was brought to the emergency room after experiencing a 20-minute episode of right-sided weakness. An MRI was performed to evaluate for a possible stroke. An axial cut of a FLAIR sequence is provided below. What is the likely diagnosis of the patient’s chronic illness?
A 28-year-old woman with a history of intractable epilepsy presents to you for outpatient follow-up. She has been seizure-free on valproic acid and levetiracetam for the last 18 months. She reports a desire to become pregnant and asks if she has an increased risk of in-utero defects with her medications. Which of the following is the most appropriate answer to this question?
A 1p/19q co-deletion is associated with which of the following tumors?